Calculating Noether Currents in Peskin & Schroeder QFT

  • Thread starter Thread starter Breo
  • Start date Start date
  • Tags Tags
    Currents Noether
AI Thread Summary
The discussion focuses on the calculation of Noether currents as presented in Peskin and Schroeder's Quantum Field Theory book. Participants express confusion regarding the derivation of the currents, specifically how they relate to the symmetry of the Lagrangian density. It is emphasized that understanding the connection between classical mechanics and Noether's theorem is crucial for grasping these concepts. One participant suggests that a solid foundation in Lagrangians and variational calculus is necessary to avoid confusion in quantum field theory. The conversation concludes with a request for specific calculations to clarify misunderstandings.
Breo
Messages
176
Reaction score
0
Physics news on Phys.org
As Peskin and Schroeder present it, the calculus is essentially that of partial derivatives while treating \phi and \partial_\mu \phi as independent variables. For a given Lagrangian density \mathcal{L}, he defines the current in eq. (2.12). However, the current depends on the symmetry at hand, which enters through the \mathcal{J}^\mu term, defined on the previous page.

If this still looks too opaque, do you remember the treatment of Noether's theorem in classical mechanics of particles?
 
Hypersphere said:
If this still looks too opaque, do you remember the treatment of Noether's theorem in classical mechanics of particles?

No, I didn't learn it in my classical mechanics subject.

My doubt is how it get those results for the currents. I tried to do the calcs but I get different results.
 
Ok... To be honest, that is a bit of a red flag! QFT is a tricky subject in its own right, but it relies heavily on classical mechanics. So you may want to pick up a good book and learn Lagrangians, variational calculus and Noether's theorem properly as soon as possible. Otherwise I think you may be in for a rough ride.

Could you show your work, say for the \mathcal{L}=\left( \partial_\mu \phi \right)^2 Lagrangian under the \phi \rightarrow \phi + \alpha transformation? That way it'll be more clear to us where your problems lie.
 
Thread 'Motional EMF in Faraday disc, co-rotating magnet axial mean flux'
So here is the motional EMF formula. Now I understand the standard Faraday paradox that an axis symmetric field source (like a speaker motor ring magnet) has a magnetic field that is frame invariant under rotation around axis of symmetry. The field is static whether you rotate the magnet or not. So far so good. What puzzles me is this , there is a term average magnetic flux or "azimuthal mean" , this term describes the average magnetic field through the area swept by the rotating Faraday...
Back
Top